Saturday, August 14, 2021

Physicist Despairs over Vacuum Energy

[This is a transcript of the video embedded below.]



Vacuum energy is all around us, it makes the universe expand with quantum fluctuations, and before you know they’re talking about energy chakras and quantum healing. Even many physicists and science writers are very, very confused about what vacuum energy is. But don’t despair, at the end of this video you’ll know why it’s not what you were told it is.

This video came out of my desperation over a letter that was published in the June 2021 issue of Scientific American. It’s a follow-up question about an article about the accelerated expansion of the universe and it reads as follows
“[The article] “Cosmic Conundrum” by Clara Moskowitz, describes how the most likely cause of the accelerating expansion of the universe is “vacuum energy,” the effect of virtual particles popping in and out of existence. But it does not explain why vacuum energy would cause the universe to expand. I would think that if space is filled with evanescent virtual particles, they would collectively exert a huge gravitational force that would counteract expansion.”
To which the editor replies:
“Vacuum energy is positive and has a constant density throughout space. Thus, increasing the volume of space increases the total amount of vacuum energy, which requires work. It is the opposite of a gas, whose energy and density decrease as it expands. When that happens, the gas exerts positive pressure. In contrast, because vacuum energy is positive, it exerts negative pressure, so galaxies on the largest scales are pushed apart, not pulled together.”
I didn’t understand this answer. Which is a little bit embarrassing because I’m one of the people quoted in the original article. So I want to look at this in a little bit more detail.

First of all, the terminology. What’s vacuum energy and why is it important?

If we leave aside gravity, we can’t measure absolute energies. We only ever measure energy differences. You probably remember this from your electronics class, you never measure the electric potential energy, you measure differences in it, which is what makes currents flow. It’s like you have a long list of height comparisons, Peter is 2 inch taller than Mary and Mary is one inch taller than Bob and Bob is 5 inch smaller than Alice. But you don’t know anyone’s absolute height. Energies are like that.

Now, this is generally the case, that you can only measure energy differences – as long as you ignore gravity. Because all kinds of energies have a gravitational pull, and for that gravitational pull it’s the absolute energy that matters, not the relative one.

So it really only becomes relevant to talk about absolute energies in general relativity, Einstein’s theory for gravity. Yes, that guy again. Now, if we want to find out the absolute value of energies, we need to do this only for one case, because we know the energy differences. Think of the height-comparisons. If you know all the relative heights, you only need to measure the absolute height of one person, say Paul, to know all the absolute heights. In General Relativity, we don’t measure Paul, we measure the vacuum.

How do we do this? For this we need to have a look at Einstein’s equations for General Relativity. Here they are. They are called “Einstein’s Field Equations”. They contain two constants, so they have the same value at every point in space and at every moment in time. The one constant, the G, is Newton’s constant and determines the strength of gravity. The other, the Lambda, is called the cosmological constant. The R’s here quantify the curvature of space-time.

And this term with the T contains all the other kinds of energies, particles and radiation and so on. This means, if we set the T-term to zero, we have empty space. You can therefore interpret Lambda as the energy-density of the vacuum. So, not the entire energy, but energy per volume. This vacuum energy-density doesn’t dilute if the universe expands because it’s a property of space-time. That makes it different from all other kinds of energy densities that we know. The other ones, for example for matter or radiation, all dilute with the expansion of the universe. The vacuum energy density doesn’t.

What does the energy-density of vacuum have to do with the acceleration of the universe? If we want to know what the universe does as a whole, we introduce what’s called the “scale factor” a. The scale factor tells you how distances change with time. So a is a function of time, a(t). If the universe expands, a increases, if the universe shrinks, a decreases. You plug this into Einstein’s equations. And then one of the equations says that the second time derivative of the scale factor, so that’s the acceleration of the expansion, as a contribution that is proportional to the cosmological constant. So that’s where it comes from. A positive lambda makes the expansion speed up.

What’s this all got to do with vacuum fluctuations? Nothing. And that’s where physicists get very confused. You see, we cannot calculate this measureable vacuum energy-density which appears in general relativity. It’s a constant that we infer from observations and that’s that.

A lot of physicists claim that particle physics predicts the vacuum energy-density, and it’s 120 orders of magnitude too large, and that’s the worst prediction ever, I’m sure you’ve heard that story. But that’s just wrong. This value which you get from particle physics is unmeasurable, so it’s not a prediction. If you hear someone claim it was a bad prediction, I suggest you ask them what theory was ruled out by the conflict between the prediction and observation? The answer is: none. And why is that? It’s because it wasn’t a prediction.

Okay, so we have learned: vacuum has an energy-density, it’s a constant of nature, it’s proportional to the acceleration of the expansion of the universe, and it has nothing to do with quantum fluctuations. This hopefully also clarifies how something that’s supposedly due to fluctuations can be constant both in space and in time. It’s because nothing is fluctuating. So that would have been my response to the question.

Let us then look at the editor’s response. This response uses an analogy between the vacuum energy-density and the simplest type of gas called an “ideal gas”. An ideal gas is just a bunch of particles moving around bumping of each other. The ideal gas has a volume, temperature, pressure and an internal energy. Internal energy is what you need to do work. The key equation is
ΔU = - p ΔV
U is the internal energy, p the pressure and V the volume. Those Δ’s mean you have small changes of the quantities that come after the delta. The pressure of an ideal gas is always positive. What this equation tells you is that if you increase the volume, so ΔV is positive, then ΔU is negative, so the internal energy decreases. This means if the gas expands it does work, and then you have less internal energy left. Makes sense.

Now, as we have seen, the energy-density of the vacuum, Lambda, is just a constant. The total energy is just the density times the volume. This mean, if the volume increases, because the universe expands, but the energy density of the vacuum is constant, then the amount of vacuum energy increases with the volume. If you identify this energy with the internal energy of a gas, this means delta U has to be positive, and if Delta V is also positive, because space expands, this can only be if the pressure is negative.

And this is correct. If you associate a pressure with the vacuum, then that pressure is negative. However, the problem with this explanation is that the vacuum energy is not an internal energy, it’s a total energy, and the vacuum energy is not a gas in any meaningful way because it’s not made of anything, and how you get from the ideal gas analogy to the expansion of the universe I don’t know.

So I don’t want to call this answer wrong, but I think it’s misleading. It strongly suggests a physical interpretation, namely that the cosmological constant is some kind of weird gas, but it doesn’t spell out that this is really just an analogy. I am picking on this because simplified analogies like this that make no sense if you think about them are the reason so many people either physics is incomprehensible or physicists have totally lost it or maybe both.

If you look at the math, the best way to think about the vacuum energy-density is that it’s just a constant of nature.

170 comments:

  1. Nicely detailed and beautifully done: Thanks!

    ReplyDelete
    Replies
    1. I spend a lot of time going squint-eyed at maths and physics pop-sci stuff and papers; it's satisfying when something comes through clear and solid like this.

      Delete
  2. I posit that chakras *are indeed* quantum phenomena: They are discrete nodes of energy concentration in one's body so they are, after a fashion, quantised. ;)
    Lucky the wind didn't change whilst Dr. Hossenfelder made that video. :)

    ReplyDelete
    Replies
    1. except that nobody can measure those alleged energy concentrations, there is no biological locus for them in the first place, and a host of other reasons why that is nonsense.

      I mean, why is the 7th chakra at the top of your head and not at the sides or towards the front, i.e. where my migraines seem to come from? Why is #4 not off to the left where the heart actually is? and so on.

      Delete
    2. Hi Smurfix,

      True - I jest though. I am intrigued that different medical and spiritual traditions insist that sone sort of force/energy is present in our bodies, so I wonder what it might be. With chakras, energy moves up and down the spine and different chakras can have blockages, is the general gist I think. The energy is characterised as Kundalini, messaging 'serpent'. The energy can have (possibly psychosomatic?) outbursts that affect people sexually and emotionally. Sorry if I'm trying to describe what you already know.

      I found out last month that a fairly common condition for young women to have is ideopathic migraines related to pressure that I have the displeasure of suffering. I'm pretty sure there's a better explanation for it than 'qi and chakras'.

      Delete
  3. Would you agree then, that if the universe were infinite and contained an infinite mount of mass, the vacuum energy-density would equal zero? (Both locally and relative to two observers separated by considerable distance?)

    ReplyDelete
  4. Vacuum energy comes from the zero-point energy of QM. The Hamiltonian for a harmonic oscillator H = ½(p^2 + ω^2x^2) written according to the operators a and a^† is H = ½ħω(a^†a + aa^†). The commutator [a, a^†] = 1 means we can write aa^† = a^†a + 1 and so

    H = ħωa^†a + ½ħω.

    That last term is the energy for zero mode or zero particles with N = a^†a. This is sometimes interpreted as a pendulum or spring that at rest has an uncertainty in position and momentum, which means it has this indelible oscillation.

    This is sometimes referred to as a quantum fluctuation. Depending on who you talk to this may be regarded as an abuse of language. It really is only a stochastic fluctuation under a measurement. Nature by herself is not really fluctuating.

    The usual argument is that in the case of a Yang-Mill gauge field, such as QED, we have a summation over all possible frequencies of photons. Then in general we can sum over all from some IR cut-off to the cut-off energy that is less than ω → ∞. The ultimate cut-off is usually taken to be the Planck scale or ω = c/ℓ_p. The cosmological constant Λ = 8πGρ/3c^2 ≃ 10^{-52}m^{-2} would indicate the vacuum energy is 123 orders of magnitude smaller than our predictions.

    Obviously something is terribly wrong. I suspect it is the following. We have two types of curvatures in general relativity, Weyl curvature and Ricci curvature, and these combine to gives the Riemann curvature

    Riem = Weyl + Ricci.

    The actual formula is a bit complicated. The Weyl curvature is a conformal curvature and it is defined according to Killing vectors with certain eigenvalues. These Killing vectors define conserved quantities. A Killing vector K_t =√(g_aa}∂_t defines conservation of energy in a Noether theorem result. In the case of g_rr for type D solution for black holes energy is conserved. The Weyl curvature defines volume diffeomorphisms that conserve the boundary area of that volume. The tides are a case of this, where the difference in gravitational field distends a body, and in the ideal limit with no resisting force the area of this body is conserved. We may think of a cloud of test masses.

    The Ricci curvature may be compared to the cloud of test masses surrounding a gravitating mass. The shell of test masses shrinks in both volume and area as they all fall inwards. There are as a result no Killing vectors or conservation laws. In the case of cosmologies they are type O in the Petrov scheme, and they have no Killing vectors or conservation laws. This is why we can have a spherical spatial surface in an accelerated expansion “eternally” which has the curious effect of creating vacuum energy from nothing.

    Most analysis is done on the Hubble frame where the Hamilton constraint Nℋ = 0 holds. This is how we get the conservation law for the vacuum

    (a’/a)^2 = H^2 = 8πGρ/3c^2.

    This has the effect of saying the kinetic energy on the left is determined by the potential energy on the right. This sounds like energy conservation, but it only holds on this preferred frame. This preferred frame is a manifestation of this solution, but it also means the theory involves “pseudo-tensors” and this is not universally valid. It holds in a somewhat delicate way.

    ReplyDelete
    Replies
    1. I meant not to comment on this because it seems rather useless, but some people seem to have gotten confused about this, so here we go.

      Lawrence is summarizing the argument that I just explained is wrong. The QFT contribution is not observable, hence it's not a prediction.

      I don't know why physicists find this so difficult to understand. I suspect it's simply because they have heard this tale so many times they cannot fathom that it might be wrong.

      Delete
    2. Just like vacuum energy itself.

      Delete
    3. I think the measurement of the Casimir effect is a prediction. The extent to which physics is solely about prediction is complicated. In fact, I seem to recall one of your videos making just that point.

      Delete
    4. Lawrence Crowell,

      "This is sometimes referred to as a quantum fluctuation. Depending on who you talk to this may be regarded as an abuse of language. It really is only a stochastic fluctuation under a measurement. Nature by herself is not really fluctuating."

      In this case, the vacuum energy for a region where no measurement takes place should be 0, right?

      "I think the measurement of the Casimir effect is a prediction."

      But is it correct to interpret the Casimir effect as a measurement of the vacuum energy, or as a measurement of the effect of the vacuum on the plates?

      To make an analogy, think about a bullet moving through water, creating vortices. The vortices only exist around the bullet. It would be wrong to assume that they exist everywhere in water.

      Delete
    5. It is not so much the vacuum energy is zero in the absence of a measurement, but that it is not this seething jumble of random stuff. There are a lot of heuristic ideas meant to explain physics that sometimes create as much confusion as they help.

      The Casimir experiment could be interpreted as a difference in energy of the vacuum between the plates as outside. As I indicated the vacuum could also be thought of as a quantum atmosphere around any object with mass and energy interacting. The two plates have quantum atmospheres that might be entangled globally, and their mutual interaction by local field theory may explain the Casimir force as well.

      Delete
    6. @ Lawrence Crowell,

      In 2005, Robert Jaffe from MIT has shown that the Casimir effect is a manifestation of QED that is unrelated to zero-point fluctuations:

      “The Casimir Effect and the Quantum Vacuum”, Phys. Rev. D 72 (2005) 021301

      Delete
    7. Lawrence Crowell,

      "It is not so much the vacuum energy is zero in the absence of a measurement, but that it is not this seething jumble of random stuff."

      But what is the vacuum energy when it's not measured?

      Delete
    8. Ervin,
      do you have an explanation or is this just an authority argument?
      Stefan

      Delete
    9. @ Goldfain: I may have to look at this. This may not be too far from what I and saying. Wilzcek wrote a paper on quantum atmospheres https://arxiv.org/abs/1805.07994 . This reproduces Casimir physics without appealing to virtual particles etc.

      Andrei: For ordinary QFT one can remove the the ZPE or vacuum energy. A process called normal ordering allows for this without damaging the physics observed. It is with gravitation that things get strange, for this does require an absolute value for energy, not just energy differences, which means such procedures are problematic.

      Delete
  5. continued: This takes us back to the question of what is meant by the quantum vacuum. Gravitation by the usual rule, “Mass-energy tell spacetime how to curve and the curvature of spacetime tell mass-energy how to move.” We must include all of the mass in a spacetime. This leads to this problem, where unlike other YM and QFTs we can normal order operators and our concern is only with the vacuum polarized or interacting with fields. The rest is irrelevant. This has led us to the potentially erroneous idea that the universe is determined by this vacuum energy everywhere.

    The Casimir effect or vacuum energy can be seen as a zero-point energy of a particle or mass. Any mass, such as the plates in the Casimir experiment have a sort of quantum atmosphere. We may think of the two plates sufficiently close so their quantum atmospheres interact to give this vacuum force. The vacuum energy of the universe may be similar. We might consider this with the classical radius of the electron ~ 10^{-13}cm, where this has its “quantum atmosphere.” We do not consider the summation of vacuum energy by the net volume in the universe defined by 10^{80} particles, but rather the area. The ansatz is that the area of this quantum atmosphere set to the classical radius is a sort of holographic surface and the net area so defined is 10^{54}cm^2. This surface contributes this mass energy with a net curvature ~ 10^{-54}cm^{-2}. This is close, or for this back of the envelop presentation not too bad.

    Of course, I can hear the howl of protest and agony now. Would this not imply that the cosmological constant varies with the spatial distribution of matter? That would be the case if this were defined entirely by local physics. Yet, if spacetime is defined by entanglement then all of these quantum atmospheres may then be in a grand entanglement and this effect is entirely then nonlocal. This is a trial balloon of an idea. It though might point to thinking about getting away from the idea of there being a constant energy everywhere in space, but rather a constant interaction energy by gravitation defined by how spacetime is built up from entanglements.

    ReplyDelete
    Replies
    1. @Lawrence: And here I was, assuming that space-time was just simply, y'know, curved.

      Delete
    2. One can presumably say that. The cosmological constant or Λ "just is," that this constant in the Einstein space has some source so that Λ = 8πGρ/3c^2. If so then this vacuum density ρ is due to some matter-energy or field source. If this is "just so," then Λ = 8πGρ/3c^2 us just a silly equation that redefines a constant.

      Delete
    3. Thank you for humouring me with a serious answer, I didn't think there was one. :)

      Delete
  6. The p in -pΔV is the pressure the gas is working against. If a gas expands into a vacuum (Joule expansion) it's doing no work so its internal energy doesn't change.

    ReplyDelete
    Replies
    1. The assumption is that the "gas" fills the entire universe. So it's not an expansion into vacuum, but work is done against other volumes of "gas".

      Delete
    2. If the volume changes the internal energy or NKΔT changes. Sabine is right technically, but the comparison with gas pressure makes some sense in a small or adiabatic limit.

      Delete
    3. Hi Werner.
      & because the assumption makes so much sense, someone will soon come up with the idea of ​​determining the distance to the edge of the universe by measuring pressure differences in a vacuum. ^.^ ,

      Delete
  7. I love this week's episode.

    "I suggest you ask them what theory was ruled out by the conflict between the prediction and observation?" I laughed at this one. Isn't it a QFT calculation? You would expect a lot of embarrassment in the professional community resulting in some novel proposals. (You have to include dark matter, axions, neutralinos, sterile neutrinos, and dark photons to get the right answer. (chuckles))

    ReplyDelete
  8. Sabine you say "However, the problem with this explanation is that the vacuum energy is not an internal energy, it’s a total energy, and the vacuum energy is not a gas in any meaningful way because it’s not made of anything, and how you get from the ideal gas analogy to the expansion of the universe I don’t know."
    Unfortunately I don't get the point, because per definition the Internal energy of a thermodynamic system means the total energy within this system at a given internal state. And if one understands the vacuum energy as an intrinsic property of vacuum then it isn't so surprising that it doesn't get diluted with volume increase (in contrast to gas).
    And I think this reasoning isn't in conflict with interpreting Lambda as natural constant. But while we have to accept the value of the gravitational constant as given I'm not sure if at least in principle the value of Lambda should be derivable from foundational principles, eg. from yet unknown quantum gravity. What do you think about that? And why fails QFT? Should't QFT be predestined to yield an answer? Sorry for lots of questions.

    ReplyDelete
    Replies
    1. What's the total energy of a proton?

      Delete
    2. The total energy of a proton is 938 MeV. (E = mc^2)
      It is important to distinguish between energy and energy density. John Baez has given a very clear discussion (math.ucr.edu/home/baez/vacuum.html)

      Delete
    3. "The total energy of a proton is 938 MeV. (E = mc^2)"

      That's the energy of the excitation relative to the vacuum. (In the restframe, absent further potentials.)

      Delete
  9. Can you explain what the cosmological constant problem is then? Does quantum field theory / quantum electrodynamics not predict a vacuum energy many orders of magnitude higher? Or are you saying that only a small fraction of this apparent energy in empty space actually contributes to the expansion of the universe (i.e. as the effect seen as the gravitational constant) ?

    ReplyDelete
    Replies
    1. There is no cosmological constant problem. It's a pseudo-problem that theoreticians go on about because they can't tell physics from metaphysics.

      Delete
  10. The accelerated expansion of the universe was discovered in 1998 by two independent projects, the Supernova Cosmology Project, and the High-Z supernovae Team, which both used distant type 1a supernovae to measure the acceleration.

    This acceleration seemed to have caught most scientists by surprise, even those taking part in the 1998 projects. For the most part they believed that the combined effect of all the matter and energy present in the universe would actually, over time, cause the expansion of the universe to slow down not speed up. But if they had taken the Lambda term in Einstein's field equations literally to be a constant of nature, it should have come as no surprise that the expansion of the universe should be speeding up not slowing down over time, as the gravitational forces tending to cause the universe to contract became weaker and weaker as their densities decreased due to the expansion, compared to the constant Lambda which is responsible for the expansion.

    ReplyDelete
  11. If we consider a virtual particle that has mass and therefore exerts a gravitational force that is located from the observer at the extreme edge of the observable universe (and is therefore moving away at a relativistic velocity of, say, 99.99999999% c), then would the magnitude of its gravitational force also increase relativistically from the viewpoint of the observer? If so, and on the understanding that its gravity would still be extremely small given the distance from the observer, would the observer see any effect of that increased gravitational force on the space very near to the virtual particle that could be interpreted as expansion?

    Maybe a simpler way to ask would be do virtual particles at the extreme edge of space create a sort of gravity well that approaches infinity due to their relativistic mass as seen from an observer and, if so, how would that affect the space nearer to the particle from the perspective of the observer?

    ReplyDelete
    Replies
    1. For one thing equal and opposite gravitational effects cancel, as Newton pointed out in his calculation that a hollow sphere would have no gravity due to the sphere inside the hollow. For another, apparent velocity due to spatial expansion does not change the effective mass of the embedded particles, as far as I know. I think only acceleration caused by forces on particles does that. For a third, gravitational effects propagate at the speed of light, so such effects that occur far, far away are not guaranteed to overcome the increasing distance and reach us.

      Fourthly, we ourselve could be considered near the edge of the universe by creatures far, far away, since from the observer's viewpoint everything is expanding away from them in all directions. Therefore what we see locally is the same as observers far away (in similar concentrations of matter) would see.

      For better, more qualified opinions, see Wikipedia or other respectable Internet sources or your local library. The people who have time to answer such questions for free on a random blog may or may not be trustworthy.

      Delete
    2. JimV,

      Thank you for the response. To respond in order...

      Yes they would cancel at the center but not distances away from the center from the perspective of the observer which is the whole point.

      Why wouldn't it? Velocity is velocity regardless of whether it's a "real" particle or a virtual particle. It's still moving relative to the observer. Relativistic mass is dependent on the velocity of the observer. For example, ultra high velocity cosmic rays are measured to exhibit relativistic properties as they slam into the atmosphere. Why would a virtual particle moving away at similar velocities not exhibit relativistic effects?

      Yes gravity moves at the speed of light, and these particles are very far away right now, but this is irrelevant because the gravity would still be arriving from the past.

      This doesn't change what would be measured by the observer.

      This isn't a random blog. And certain answers on here I would find highly trustworthy.

      Delete
    3. As the hollow earth calculation showed, forces cancel not just at the center but everywhere in the interior.

      Relativistic mass is the effect that it becomes harder (takes more energy) to accelerate a mass the closer it comes to the speed of light. This does not apply to the expansion effect on velocity, which has no such limiting effect.

      Gravity from the past is already here and has little effect on our local gravity. All of Newton's thousands of calculations of astronomical observations such as orbits of Saturn's moons were based on local masses only and agreed precisely with observation. (Source: James Gleick's biography of Newton. See also Simon Singh's "The Big Bang".)

      Answers in a blog comment without much in the way of math or exercises to work through that math are only going to provide a very shallow depth of information. (Note that you apparently assumed Newton's calculation only applied at the center since you had not looked up the calculation or worked through it yourself.) Asking the experts to provide you with quick answers is rather an unjustified use of their time, in my opinion. Finding out for yourself from in-depth sources and personal effort would be better for both yourself and the experts.

      It's really none of my business and I should not have made any reply, but I feel sorry for the experts. In my experience one cannot educate everyone or even a lot of people by answering random questions unless they work hard at it themselves. Anyway, I've done my best from what I know and the rest appears to be up to you.

      Delete
    4. The hollow earth calculations you are referring to are based in Newtonian dynamics and do not take into account general relativity which is nonlinear near the boundary. This distinction is the principle subject of my original post.

      No. Relativistic mass is not restricted to increasing the energy required for further acceleration. It is actual, calculable mass that exerts actual, calculable gravity. If you are trying to say that general relativity distinguishes between a particle’s relativistic velocity from moving through space as opposed to moving with space, then I don’t see a basis for that conclusion or distinction here. A particle at the far edge of the visible universe would be measured by an observer as moving through space at nearly the speed of light regardless of the contribution from expansion and regardless of the vector away from the observer.

      What? Just as light is arriving from the edge of the visible universe that was emitted ~13B years ago (or from a location those light years away), gravity is arriving from particles at those same locations. As an aside, obviously Saturn is not a significant distance from an observer if measured within a sphere the size of the visible universe. Thus, I’m not sure what you are trying to say here.

      Yes I think you may have become stuck on Newton and decided to criticize discussion - as though all of these things are known for certain if one only opened a book. But for the most part they aren’t known for certain, which is why these discussions (thankfully) still occur in the first place.

      Delete
    5. If we consider a virtual particle that has mass and therefore exerts a gravitational force that is located from the observer at the extreme edge of the observable universe (and is therefore moving away at a relativistic velocity of, say, 99.99999999% c)

      Relativistic mass is not real. Particles moving fast do not have greater mass and so do not exert more gravitational pull.

      Delete
    6. If you re-read what you wrote and compare it to my response, you should see that I was giving counter-examples to your assertions. Example: "velocity is velocity"; no, velocity due to force and acceleration has a limit, velocity due to spatial expansion does not--they are distinguishable.

      Here's a last one: Dr. Alan Guth in his book "Inflation" uses the hollow Earth example to explain some of his ideas on Cosmic Inflation. I think if General Relativity refuted Newton's calculation significantly he would have mentioned it, which he did not. I also think that if there is another GR effect at the boundary, it would not apply to local effects here due to masses far away ("at the edge of the universe").

      My hope was, new facts might cause a change in opinions.

      I see another commenter, Scott, also disagrees with your premise, as I did in my first response, albeit at much more length. Here is an experiment you might wish to fund to check your premise: establish a large laser beam passing the end-mass of a long pendulum on one side and none on the other side, and see if the relativistic mass of the photons exerts gravity to bend the pendulum towards that side. (First do a search to see if similar experiments have already been done.)

      Delete
    7. JimV,

      Photons have only relativistic masses and they should gravitate. Consider the following experiment:

      Place equal quantities of matter and antimatter in a box with mirrors on the inside. If photons don't gravitate the mass of the box should decrease when the content annihilates. This doesn't sound right.

      Delete
    8. @Scott

      They very much do when the motion contibutes to the overall energy of a system (or the system’s internal energy). The most obvious example is the mass of a proton which is much greater than the rest masses of its component quarks due to the kinetic energy of the quarks. In fact, if you were correct there would be no such thing as invariant mass. An observer and an observed particle together always constitute a system for which an increase of their relative velocity increases their collective mass. (For example, earth plus nearby cosmic ray moving 99% of the speed of light weighs less than earth plus nearby cosmic ray moving 99.999% of the speed of light.) Incidentally, the “nearby’s” in that sentence are optional.

      @JimV

      I don’t know why modern books would be based on Newton because it isn’t how the universe works.

      @Andrei

      Yes the internal energy in the box is the system and the energy post annihilation still accounts for the rest masses of the pre-annihilation particles.

      Delete
    9. I apologize for mistakenly thinking that photons had no gravity. Later it occurred to me that GR works on energy density and of course photons have that. I stand by the rest of my statements. Newton's theory can be derived from GR as an approximation, and was what NASA used to get to the moon and back. It is very accurate locally, and that alone should disprove effects from far-away, expansion mechanisms. As I understand it, the GR equations are very hard to solve unless symmetry is assumed, and Newton's Theory is still widely used, at NASA and for most engineering calculations. You can get a Masters in Mechanical Engineering without any knowledge of GR, but a thorough familiarity with Newton's theories is required. Go to any college bookstore and peruse the text books for physics and engineering. You will find Newton all over the place, and rarely find an undergraduate text that uses GR. Classical Mechanics was all Newton when I took it.

      As a final example, the calculations of galactic rotations in astronomy are all done using Newton. See the Triton Station blog for examples.

      Earlier I think you mentioned cosmic rays as somehow supporting your premises? If expansion had any effect on them it would be to slow them, since the ones we see are moving toward us, and expansion tries to move everything external away from us.

      Delete
  12. I'm having trouble understanding your paradigm, whereas I understand the ordinary paradigm well enough.

    According to the ordinary paradigm, quantum fields have vacuum fluctuations, which creates a constant energy density, which will contributed to accelerated expansion if it's positive.

    What part of that chain of reasoning do you object to?

    ReplyDelete
    Replies
    1. Mitchell,

      It's not a "paradigm", I'm just telling you what the math actually says. Lambda is a free constant in GR. If you add a constant to a free constant it's still a free constant. You go and measure it. And that's it. You could calculate 80 thousand QFT contributions to 90 digits after the point and add them, and it'd still be a free constant that you have to go and measure. Because the contribution from QFT is unmeasurable by itself.

      Delete
  13. I've already downloaded a PDF explaining the Laws of Thermodynamics to read and learn at my leisure, but can someone please explain what 'internal energy' and 'total energy' are in this context, thanks?

    ReplyDelete
    Replies
    1. The internal energy is that of an ideal gas. In an ideal gas there are no forces between particles hence the only energies are the kinetic energy of the particles. Thus the internal energy of an ideal gas is the total kinetic energy.

      In contrast, Sabine post describes the total energy as the total energy of the vacuum. This is the product of the vacuum energy density by the volume.

      Now, the vacuum is not modelled as an ideal gas. Hence we can't identify this total energy as the internal energy of a gas. But if we were to do so, then the pressure of this has would be negative in contrast to a normal idwal gas whose pressure is always positive.

      Personally, I think that given LQG description of spacetime via spacetime atoms and hence of the vacuum by spacetime atoms, then one might think that this spurious analogy might not be so spurious after all.

      Delete
    2. Ah, thanks Mozibur. I didn't quite put 2 and 2 together.

      Delete
  14. The editor's explanation sounds a bit like an explanation I heard from Max Tegmark, so maybe that's where he got it.

    ReplyDelete
    Replies
    1. I contacted the editor and she told me where she got it from and I also contacted the person she named just to make sure I interpreted it correctly. It wasn't Tegmark.

      Delete
  15. Is there any connection between this vacuum energy density and the inflationary paradigm, which explains that our universe was the result of the decay of the inflaton field which rolled down the hill to its minimum value, and this minimum value happens to be the vacuum energy density that is named the cosmological constant?

    ReplyDelete
    Replies
    1. Yes, in that during inflation the vacuum energy was far larger. it would have been close to that 122 orders of magnitude difference. In the eternal inflation scenario the de Sitter spacetime of inflation was close to the Planck scale in vacuum energy density, and there was a transition through a tunneling barrier. This generated a bubble that was not at the minimal vacuum configuration. The transition to that configuration meant the mass-gap from high energy to the low-energy physical vacuum generated radiation and matter.

      Delete
    2. Abd as a follow up question, is there any ongoing experimental effort to find the inflaton particle, or is the energy level at which we could find it too far out for current detectors to probe?

      Delete
    3. I am not sure if there is an inflaton particle per se. There is a Ginsburg- Landau-like potential, which is similar that for the Higgs field. Yet, if there is a particle it is near Planck mass. We really might not want to excite modes of such a particle. We might cause an inflationary to emerge.

      Delete
  16. Sabine, in the video you seem to imply that Lambda is just another constant appearing in Einstein's equations, on a par with Newton's constant G. I know this is the opinion of some people, like Rovelli, who even wrote a paper about it (https://arxiv.org/abs/1002.3966).
    May I disagree? Without Lambda, Einstein's equations still make perfect sense (they've been used like this for decades). In contrast, without G, Einstein's equations become meaningless as a theory of matter and spacetime (the two decouple), and so does Newtonian gravity with G=0 for that matter.
    This would seem to suggest that the two constants do not have the same status, with G being fundamental to the theory, while the value of Lambda is the result of some accident (no idea which).

    ReplyDelete
  17. As to that 120 orders of magnitude "error". It does negate one thing: the "zero point energy" you get from theStandard Model does not contribute to the stress-energy tensor.

    Of course, in totally unbroken supersymmetry, that energy is exactly zero. But there is still no reason to expect from that that the cosmological constant would be zero. So it does not negate unbroken supersymmetric string theory either.

    ReplyDelete
  18. That's just shifting the problem somewhere else: where does the contribution come from that exactly cancels the QFT part?

    ReplyDelete
    Replies
    1. As I said, it's a free constant in GR. You never measure the QFT part alone. You can't. It's unobservable. That physicists go on about a math thing which is by construction unobservable is one of the reasons my book is called Lost in Math.

      Delete
  19. It seems natural to me that there should be the ability to observe the action of the cosmological constant in action here on earth. Since we are convinced that the cosmological constant exists and results in the production of energy from nothing and its creation results in the accelerating expansion of the universe, we should be able to see the effects of the cosmological constant manifest locally and observe its presence through the production of energy from nothing here on earth in the lab. If this energy can propel entire clusters of galaxies beyond the speed of light, it should have a measurable impact somewhere here on earth. Furthermore, the majority of the universe is composed of this energy. But there is also dark matter and dark energy which also seems to be an integral and inextricable component of space-time. From the very beginning of the universe, the percentage makeup of the universe: vacuum energy, dark energy, dark matter, and bright matter has not varied. Looking back billions of years in time we see the same look and behavior in every aspect of the universe that we see locally today. Not observing this energy, and the associated creation of both dark energy, bright, and dark matter here locally must involve the way that we are looking for it. Or could it be that we are fearful of seeing it?

    ReplyDelete
    Replies
    1. If this energy can propel entire clusters of galaxies beyond the speed of light, it should have a measurable impact somewhere here on earth.

      This is a misapprehension. Galaxies are not being 'propelled' through space by the expansion of the universe. The universe is simply expanding.

      Delete
  20. This comment has been removed by the author.

    ReplyDelete
    Replies
    1. Hang in there J.C. I'm 78. After following Backreaction for a couple of years I'm already up to understanding maybe 5%.

      Delete
    2. I was thinking yesterday, I'm participaing in discussions here with some members of the world's physics community, albeit on a microscopic level. That feels both mind-blowing and a bit ridiculous, given my pretty miniscule understanding of most topics at hand.

      Delete
    3. If there is one thing that I'm pretty certain about now it's that, Lambda is a free constant.

      Delete
    4. And adding stuff to it doesn't change that.

      Delete
  21. The ultimate causation of the cosmological constant, the creation of dark matter and energy, and the ongoing production of bright matter is most likely rooted in a local space-time generation process that has at its heart spontaneous symmetry breaking. Conservation laws are a result of symmetries. It is apparent that there is an ongoing locally originating dynamic space-time creation process that violates symmetries on a wholesale basis so that most conservation laws are violated as an ongoing local mechanism within each and every galaxy throughout the universe that leads to the generation of both bright matter, dark matter, and dark energy within each and every galaxy.

    The need for a local solution to maintaining constancy in a meriod of critical universal mechanisms is rooted in the limitations in the speed of causality. A well ordered universe is driven by an all inclusive control process that is locally actioned.

    The matter-antimatter asymmetry problem is a result of this dynamic space-time creation process where spontaneous symmetry breaking occurs continually everywhere throughout space-time.

    Every galaxy adds space-time to its initial allocation which acts to push its neighbors outward in all directions. This new increase in space-time includes but is not limited to vacuum energy, the Higgs field, dark matter, dark energy, and additional matter increase.

    This space-time expansion mechanism has maintained a flat space-time from the earliest moments of the expanding universe. This wide flexibility in adapting to major changes in the evolution of the universe speaks against the primacy of initial conditions and a fixed cosmological constant as a determining factor in the development of the cosmos. The universe has undergone many stages of evolution and countless phase transitions but the space-time expansion mechanism has kept the universe flat and the laws of nature fixed. This flexibility in maintaining constancy is witness to a dynamic and complex adaptive process that is driven by multifaceted feedback from the real time status of the universe at every nanosecond of its existence.

    ReplyDelete
  22. So the scale factor is accelerating. We are living in a transitory period of an unsustainable universe that will go pop.
    Although prof Subir Sarkar / Oxford / suggests we consider that lambda may be zero and we are fooled by the bulk flow.

    ReplyDelete
  23. So, observations do not agree with theory. Feynman had some comments about that.

    https://www.youtube.com/watch?v=OL6-x0modwY#aid=P20kDjskZwA

    That the Universe is expanding is based on one single premise: the Hubble Red Shift is due to a Doppler effect recessional velocity.

    When Hubble published his observations of red shifted light from distant objects there were two possible explanations that came to the fore. One, originated by Georges Lemaitre, was that the Universe was expanding. The other, from Fritz Zwicky, was that light lost energy as it traveled, termed "tired light". At that time, ca, 1930, interstellar and intergalactic space were assumed to be perfect vacuums, or at least close enough for government work, and thus there was no mechanism to redden the light.

    Now, 90 years later, we have a much better understanding of reality. But more importantly, far more importantly, we have actual observational evidence that Zwicky was right.

    In the radio astronomy of Pulsars we find that the shorter wavelengths of the leading edge of the pulse arrive before longer wavelengths. The velocity of light, c, is NOT constant but varies by wavelength. The conventional explanation is that the DM is the “integrated column density of free electrons between an observer and a pulsar”.

    https://astronomy.swin.edu.au/cosmos/P/Pulsar+Dispersion+Measure

    The mechanism matters not. What matters is that the interstellar medium is not a vacuum but rather affects light waves in a way best described as having an Index of Refraction greater then 1, unity. We find the same phenomenon in the observation of Fast Radio Bursts from other galaxies, thus indicating that the intergalactic media is not an electromagnetic vacuum.

    Conclusion: The velocity of light, c, is not a constant. The Hubble red shift is the result of the light traversing a distance through a medium denser the Eintein's "in vaccuo" rather than a recessional velocity.

    If the universe is not expanding it explains a few things.

    ReplyDelete
    Replies
    1. Hello all,

      has anyone read Roy's contribution?!
      (about the redshift and the dispersion of pulsar pulses).
      Can someone explain it to me?
      Maybe you Sabine? Or you Lawrence? Or you Seven Evans? Or anyone else?

      Stefan

      Delete
    2. Stefan,

      To make a long story short, it's a fringe idea that fits badly to the data. I've debated with myself whether to even publish this comment.

      Delete
    3. Stefan and Roy have fallen into the same trap that Zwicky did. Zwicky made an assumption the belied all conclusions (even to this day) - that a decrease in (potential) energy would somehow increase the oscillation period (??). In fact, photon decay, like every other oscillation, would require a decrease in the period and a subsequent increase in frequency, resulting in a blue shift. The erroneous mindset is because our budding physicist are taught that ultraviolet light has more (potential) energy than infrared. Educators gloss over the fact that a single photon of light can knock one electron from a metal surface while a single photon of radio frequency can slightly modify trillions of electrons over the same slice of (quantified) time.

      Einstein looked long and hard for photon decay. This was the seeming delay that he and Hubble were extending between the announcement of galactic red shift and their endorsement of expansion.

      Einstein knew full well that there could only be two possibilities and if the universe was not expanding, then it had to be infinite to make gravitational red shift a reality. Even without galactic rotation staring him in the face like later theorists, he couldn’t endorse infinity because of the white light catastrophe (for the night sky to appear as a black body, the photon would have to decay). Alas – no decay – no depletion of light - and no infinite universe.

      Even though a blue shifted oscillation decay would neatly explain galaxy rotation problem, we still balk at infinite universe theory. The currently accepted average density of the universe is a least one magnitude less than that predicted during the 1940’s. However, for blue shift to explain both rotation and black body, there would have to be period loss (in visible light) of 10 to 100 nanometers per 1 to 10 billion years. Our current technologies could detect this if we were to, say, place a ULF generator on mars. Unfortunately, this is still considered fringe. Granted, we occasionally look for the corresponding oscillation decay in protons but, honestly, if we cannot detect decay in photons, what would be the point of looking at protons?

      Not trying to convince anyone here.
      Just correcting the underlying concepts so that Roy can rephrase his thought without the woo that Zwicky started.

      Delete
    4. Gerald,

      Again with the theories! Actual observations show clearly that photons of longer wavelength/lower frequency/lower energy travel more slowly.

      Delete
    5. Roy,

      Like all theorists working in this area, I too have gone down that rabbit hole and mulled it to death. Even if you allow for the presence of an aether, there is no compelling reason for massless objects to lose velocity. In our macro world you can show that a water ripple and a water wave travel at different speeds but not by much and entirely due to lateral friction. You could argue that there is lateral friction in aether but now you not only have to convince people that there IS an aether but that the magnitude of friction steps through the Planck length.

      It’s the initial input of energy generating the wave that gets people confused. They are under the impression that a single atom can generate a radio wave because a single atom can do it for the spectrum of light. Just as it is the combined effort of trillions of atoms to collapse a radio photon into the electrical potential of the antennae, so the combined effort of trillions of atoms in the generation of the photon. i.e. Are you dropping a large rock or a small pebble into the water? (and what are the amplitudes?)

      You will note that the most controversial thing I said was that all frequencies have the same energy (potential). The really startling thing about that statement? If blue shift is detected, then longer wavelengths have MORE potential energy than shorter wavelengths. It solves the whole entropy problem but there would be much wailing and gnashing of teeth.

      Delete
    6. Regarding: Roy: " Conclusion: The velocity of light, c, is not a constant. The Hubble red shift is the result of the light traversing a distance through a medium denser the Eintein's "in vaccuo" rather than a recessional velocity."

      This observation about "tired light" is a certain indicator that Dr. H is correct about the superfluidity of dark matter and more broadly the coincident occurrence of wide spread Bose condensation(BEC) in dark matter through space. It is well known in condensed matter science and quantum physics that light is slowed down in a Bose condensate. Research into this praenomina has even shown that light can be made to stop in a Bose condensate within a crystal. See:
      https://science.sciencemag.org/content/295/5553/255

      By use of a Bose--Einstein condensate, Danish physicist Lene Vestergaard Hau (Harvard University) succeeded in slowing a beam of light to about 17 meters per second.

      Since space contains mostly dark matter, light should be expected to be slowed by multiple BEC formations of dark matter formation spreading over gigaparsecs.

      Detecting slowed light might provide a new tool in mapping coherent dark matter throughout the universe.

      Delete
    7. Light is of course not slowed down in superfluid dark matter, that would be incompatible with observations.

      Delete
    8. It could be that dark matter affects the velocity and structure of light via an unusual gravitational mechanism.

      The issue is how to explain the observation of slowed light is free space?

      It may be that the BEC of dark matter gravitationally effects free space in such a way that the portion of space that the light traverses is masked in a manner that permanently affects the group velocity of the beam of light.

      This process of masking has been seen in experiments on various beams of light.

      https://arxiv.org/abs/1411.3987
      Photons that travel in free space slower than the speed of light


      It has generally been thought impossible for particles of light, known as photons, to be slowed as they travel through free space, unimpeded by interactions with any materials.

      In a new paper published in Science Express today (Friday 23 January), researchers from the University of Glasgow and Heriot-Watt University describe how they have managed to slow photons in free space for the first time. They have demonstrated that applying a mask to an optical beam to give photons a spatial structure can reduce their speed.
      The team compares a beam of light, containing many photons, to a team of cyclists who share the work by taking it in turns to cycle at the front. Although the group travels along the road as a unit, the speed of individual cyclists can vary as they swap position.

      The group formation can make it difficult to define a single velocity for all cyclists, and the same applies to light. A single pulse of light contains many photons, and scientists know that light pulses are characterized by a number of different velocities.

      The team's experiment was configured like a time trial race, with two photons released simultaneously across identical distances towards a defined finish line. The researchers found that one photon reached the finish line as predicted, but the structured photon which had been reshaped by the mask arrived later, meaning it was travelling more slowly in free space. Over a distance of one meter, the team measured a slowing of up to 20 wavelengths, many times greater than the measurement precision.

      The work demonstrates that, after passing the light beam through a mask, photons move more slowly through space. Crucially, this is very different to the slowing effect of passing light through a medium such as glass or water, where the light is only slowed during the time it is passing through the material -- it returns to the speed of light after it comes out the other side. The effect of passing the light through the mask is to limit the top speed at which the photons can travel.

      If the dark matter BEC is affecting space to slow light, then this effect can provide a clue to the nature of dark matter.

      Delete
    9. Regarding: "The velocity of light, c, is NOT constant but varies by wavelength. The conventional explanation is that the DM is the “integrated column density of free electrons between an observer and a pulsar”."

      According to Roy's posit, free electrons affect the space that light travels through. I would like to understand what free electrons in space affect,  and how free electrons  affect  that component of this light transport medium.

      Or is the reference cited in Roy's post making an assumption by astronomers about what affects a light pulse's travel through space.

      In my posit, the structure of the pulse changes with no influence on frequency.

      To the best of my understanding in doppler shift, the spectral lines of any given element are all shifted by the same amount regardless of the frequency of each spectral line.

      Delete
    10. Roy,

      . . . or the entire universe steps slightly to the right and 4*10^50 quarks continue an exchange to the left.

      (tongue-in-cheek but deadly serious)

      In a finite universe, this creates a logical inconsistency for general relativity. In an infinite universe, it has no consequence.

      Delete
  24. Sabine,

    String theorist 4gravitons has a blog post about the QFT calculation of the vacuum energy:

    Zero-Point Energy, Zero-Point Diagrams

    https://4gravitons.com/2020/09/04/zero-point-energy-zero-point-diagrams/

    If I understand him correctly, one cannot actually calculate the energy of the vacuum itself. He says:

    “A zero-point diagram, then, means zero particles in, zero particles out”

    And:

    “Remember, each Feynman diagram answers a specific question, about the chance of particles behaving in a certain way. You might wonder, what question does a zero-point diagram answer? The chance that nothing goes to nothing? Why would you want to know that?

    To answer, I’d like to bring up some friends of mine, who do something that might sound equally strange: they calculate one-point diagrams, one particle goes to none.”

    He then explains that what is actually calculated is the energy in the close proximity of some material object (like the plates used to measure the Casimir force).

    Do you agree with this point? And if so, would you agree that the calculated value should only be used for those locations where the calculation makes sense (near material objects)? Is there any justification to extrapolate this calculation to intergalactic space?

    In your reply to Tbeck you say:

    “You never measure the QFT part alone. You can't. It's unobservable.”

    This seems contrary to the usual interpretation of QM, that it gives predictions for measurement results. If there is no experiment that could, at least in principle, measure that quantity then how would you interpret this QFT calculation?


    ReplyDelete
    Replies
    1. Anredi,

      You are mixing up two different things. The sentence you quote from me was referring to the alleged QFT contribution to the vacuum energy, that's a constant. I am saying you can't measure that.

      You can very well calculate and measure how quantum fluctuations (which is a fancy word for certain diagrams or integrals) locally change the properties of the vacuum, see Lamb shift or Hawking radiation. You do not in these cases actually calculate the absolute energy of the vacuum. This always remains an additional constant. Well, actually it's infinite. The entire reason physicists discarded this infinity was by saying you can't observe it anyway. 50 years later they'd managed to find a better way to get rid of the infinity and had forgotten it's unobservable.

      To make a long story short, I think there's no disagreement among physicists about what the math says. There's on disagreement about what it means.

      Delete
    2. Sabine,

      "You can very well calculate and measure how quantum fluctuations (which is a fancy word for certain diagrams or integrals) locally change the properties of the vacuum, see Lamb shift or Hawking radiation."

      OK, but we can interpret those calculations in at least two different ways:

      1. Quantum fluctuations are an intrinsic property of the vacuum everywhere, and this property is passively revealed by looking at atomic energy levels (Lamb shift) or event horizons (Hawking radiation).

      2. Quantum fluctuations are a property of matter/vacuum interface. They only arise when atoms and black holes are present. Vacuum in itself, in the intergalactic space does not fluctuate.

      If 1. is true we can say that those fluctuations contribute to the cosmological constant. If 2. is true, they do not contribute at all. What is the math implying? Should we assume 1 or 2 (or some other option)?

      Delete
  25. Dr.Hossenfelder
    Even if we interpret Lambda as a constant of nature (without connecting it with the vacuum energy),it seems that we cannot avoid other implications that the Cosmological Constant, combined with QFT ,has. For example, the Gibbons/Hawking entropy (and the associated temperature) of a De-Sitter cosmological horizon.
    Our universe will asymptote DS in the far future, so these notions are not of academic interest only, yet there is no consensus about their physical interpretation.

    ReplyDelete
  26. ZERO-POINT... Not vacuum energy...

    DCN

    ReplyDelete
  27. To elaborate a little (my previous comment):
    In the far future, our universe will approach, asymptotically, De-Sitter, when the horizon area reaches its maximum and then
    this area depends on the value of Lambda only. Yet, judging from the papers I've read about the subject, there seems to be no consensus at all about what this entropy means, physically.
    Is it just a geometric property of the horizon itself, or is it associated with some other degrees of freedom?
    The DS temperature, also, seems to be undetectable, even in principle! The average wavelength of the Gibbons/Hawking photons has the same order of magnitude as the DS horizon itself.
    Moreover, it seems impossible to built a detector as big (from what? There's almost nothing in a universe that approaches DS), and that isn't a practical problem only!
    Ok, these issues are only marginally connected with the post/video content,but they exist, whatever we think Lambda really is.

    ReplyDelete
    Replies
    1. It's funny to think that the maximum entropy of our observable universe is just a finite, constant number (depending only on the C.C. value), regardless of the matter content (black holes, normal or dark matter/radiation)that it had in any moment of the cosmic time during its history.
      This is, maybe, not that innocuous as it seems at first glance:It has been shown that, as the universe expands, for every galaxy (with the millions of black holes it has), that crosses the cosmological event horizon,there is an expansion of the area of the horizon, that compensates (more than enough) for the entropy loss (mostly of the receding galaxy's black holes).
      If the generalized 2nd law holds, this, maybe, implies that the entropy of the cosmic horizon and that of each black hole must have the same interpretation, otherwise an inconsistency may occur, so the strong Holographic Principle (that associates the black hole horizon area with some internal degrees of freedom), seems doubtful, as some QG people suggest.

      Delete
  28. Sabine -- would you be willing to react to Sean Carroll's 2019 twitter post below? Are you saying there is no puzzle, no conundrum, no vacuum catastrophe?

    Jan 12, 2019
    @seanmcarroll

    16. Dark energy is not a new force; it's a new substance. The force causing the universe to accelerate is gravity.

    17. We have a perfectly good, and likely correct, idea of what dark energy might be: vacuum energy, a.k.a. the cosmological constant. An energy inherent in space itself. But we're not sure.

    18. We don't know why the vacuum energy is much smaller than naive estimates would predict. That's a real puzzle.

    ReplyDelete
    Replies
    1. That's correct. There is no puzzle, there is no conundrum with the cosmological constant, it's a pseudoproblem. It's a confusion between physics and metaphysics. The value of a constant you cannot observe is not a problem for physics.

      And please note that I chose my words carefully. There is a problem with vacuum contributions, but it's not the constant mean value, it's the fluctuations around the mean value. This, however is NOT what physicists usually claim is the problem.

      Delete
  29. What I mostly took away from this discussion is the genius of Einstein 's General Relativity is similar to Newton's Mechanics in that they were rare, huge leaps in our understanding of physics yet they also have flaws we try to explain away within their dynamics. Like we eventually did with Newtonian mechanics need to move on from GR, take what it taught that is undeniable, apply it to the foundational axiom's we've ignored (do to physics math's we religiously cling too), and start again from the bottom up; or we can continue to explain away these issues for another 300 years with unseen dust and gravitational influences like we did for Newton...

    ReplyDelete
  30. Dr. Hossenfelder,
    An interesting choice of concepts as I have started thinking about vacuum energy as part of what I think is a related possible concept, mass and matter production throughout the universe. Given the close relationship between mass and energy as established by Einstein, if we are to believe that energy is not conserved because there is a constant influx of energy in order to maintain the vacuum energy density it seems to me to follow that there must also be a constant influx of mass/matter into our universe. To me the interesting thing about both of these concepts are all of the questions that need to be answered as there really are so many. If there is an influx of energy and mass, then where does it come from and how is it infused into our universe? Similarly, how would we be able to see and understand this infusion of energy and mass? Stated another way, what would need to be in place for this infusion process to occur? These are just the broadest question and each one of them has a bunch of associated questions.

    It seems to me that it would be a bit easier to find and then ultimately understand mass/matter infusion because we have a clue of what to start looking for, quarks as they are the basic building blocks of all matter in our universe. Of course now the question is how would we see or find quark pairs (real ones rather than virtual ones) popping into our universe? A thought for consideration if there is mass/matter infusion and we can better understand it and/or find it, it may lead to answers of questions associated with vacuum energy.

    I believe that there is a lot of related evidence and physics for looking at trying to find mass/matter production in our universe. And, I believe the evidence has been around for years. But because it does not further the current areas of accepted research, or it creates a problem for current research, it is ignored in hopes of being explained at a later time. All of this begs another question, if a concept, theory or hypothesis is a problem now in the building phase of modern physics, why would something discovered later be able to solve this problem?

    Just some "food" for physics thoughts.

    ReplyDelete
    Replies
    1. @Steve Guderian


      Regarding: "I believe that there is a lot of related evidence and physics for looking at trying to find mass/matter production in our universe. And, I believe the evidence has been around for years. But because it does not further the current areas of accepted research, or it creates a problem for current research, it is ignored in hopes of being explained at a later time. All of this begs another question, if a concept, theory or hypothesis is a problem now in the building phase of modern physics, why would something discovered later be able to solve this problem?"

      Steve, if you are willing to take this matter into your own hands, you can begin with an experiment that any kid can perform.


      I described this experiment in this post in the thread: "Does the Universe have Higher Dimensions? Part 1"

      with link as follows:http://backreaction.blogspot.com/2021/04/does-universe-have-higher-dimensions.html?showComment=1618288437895#c7343104230521498082

      If you undertake this experiment, keep the experiment as contamination free as possible.

      You should see a crenellated ball of layered elements produced that are different from any in the experiment. Locate it from the ash using a magnet because it contains iron. This type of space-time matter output is what you are interested in. 

      Delete
  31. This comment has been removed by the author.

    ReplyDelete
  32. This comment has been removed by the author.

    ReplyDelete
  33. In March, 1, 1896 the French physicist Becquerel accidentally discovered an entirely new phenomenon. He left a package of uranium salts on top of a roll of photographic plate in a drawer, and days later found that the plate had darkened as if it had been exposed to light; he thought that these salts had emitted penetrating rays that were able to pass through metal. However, Becquerel lost interest in the topic and passed it on to a Polish student who hadn’t yet chosen her doctoral thesis topic. She, Marie Curie saw the prospects implicit in this discovery and thereupon decided to investigate more thoroughly these stones that constantly emitted so much energy and didn’t seem to be consumed, and she baptized this phenomenon as radioactivity.


    It was just a few years later that Leo Szilard patented a method  utilizing a  chain reaction among atoms to access the energy content of radioactivity  that eventually came to be known as nuclear fission. In the face of great need, Szilard convinced Einstein to use his prestige to motivate the US government to create a project to  engineer a process to extract that energy from the atom. 

    In this time, in our present time of great need; it's now; it's now  the proper time for another Leo Szilard to arise, to inspire another transformative project to utilize current scientific discoveries in vacuum energy to extract that energy and utilize it  to resolve and remediate our many energy and environmental conundrums. But there is great fear at the danger involved in such a discovery because this method would turn the world on its head. Society along with the vested interests that support and control it would be ripped apart. The implementation and subsequent  utilization  of the method  and its repercussions to society would be utterly disruptive and profound. 

    But don't be sanguine in the immensity of the effort. Don't think that this vacuum energy extraction process is impossible. Far from it. ASHOKE SEN  has shown us the road to the realization of  such a method. Sen has calculated that the destabilization of space-time by its antithesis is possible. In the same way that antimatter transforms matter into energy, a micro bubble of anti space time will convert a micro bubble of space time into the energy that it contains. The method simply involves fashioning a bubble of anti space time and driving that bubble into a state of instability whereupon it explodes both itself and its opposite partner. 

    The issue in turning the method into reality is the will, the fortitude  and the courage to begin. Who has the will and the courage to change the world and apply existing  knowledge and discovery to run our current reality through the reality grinder to advance and improve the prospects of humankind. Are the forces arrayed against this noble effort too awesome to overcome? Is there anyone willing to make the sacrifices necessary to proceed? Who can stand up to the pressures, propaganda,  and ridicule  that the special interests will apply to retain their investments in the status quo ...Anyone?

    ReplyDelete
    Replies
    1. Hi Axil,

      I'm skeptical about the method you describe (how much time, money and effort will research and development tske, versus using methods we already have?) but I think we need something to pull the rug out from under our societal hegemonies that ultimately serve a very few. We need access to affordable, sustainable energy for everyone on Earth than we do grotesquely rich men bankrolling space projects, and the pressure is on countries that are holding back to move on from fossil fuels.

      There are currently technologies that are both established and in development that harness renewable energy and are proving to be reliable. I watched a program in The last couple of days that focused on renewable energy projects in Australia.

      I see that unless the Australian and other hold-back governments fully n back renewable energy and climate change mitigation, mass protesting, social disorder and mass distribution are inevitable.

      Delete
    2. 2/ If push comes to shove, I hope I can find a way to join that fight.

      Delete
  34. I am sorry if I've missed it somewhere in the thread, but why excactly is (absolute) QFT vacuum energy not observable in principle?

    I mean, I agree that it is not observable withtin the QFT itself, but why it cannot be atributed as a source of Lambda? and measured by measruing Lambda?


    thanks

    ReplyDelete
    Replies
    1. Lambda is a free constant. If you add a constant to it, it's still a free constant.

      Delete
    2. It is a free constant indeed within(!) the GR. The same way as the pion mass is a free constant within the theory of pions, but it is not (at least in principle) in the QCD. Isn't it?

      In other words, when we try to combine the GR and QFT, and both have Vacuum energies, it is only natural to assume that they should be the same.

      Delete
    3. "In other words, when we try to combine the GR and QFT, and both have Vacuum energies, it is only natural to assume that they should be the same."

      I have no idea what makes you think so and don't know what you mean by "natural". And no, it is obviously not the same as with the pion mass because we don't derive GR from QFT.

      Delete
    4. This paper says that the QFT vacuum is directly includable in the EFE but not at zeroth order. Lambda is a measurable consequence of it.

      doi: 10.20944/preprints201901.0113.v4

      This paper results from our investigation into novel means of electromagnetic propulsion. It requires the basis of our claims to be put on a sound theoretical footing regarding the purported momentum exchange with the electromagnetic field. One of these concerns is the huge discrepancy between the energy density of the Zeropoint and its purported manifestation as the Cosmological Constant. Here we state that it is manifestly wrong to introduce the zeropoint at zero order into the stress-energy tensor, because it is something which describes zero particle count. As a fluctuation, it belongs in a higher order Taylor expansion in frequency of the stress-energy tensor. Furthermore in the 3rd order in the Einstein constant our procedure is some 9 orders of magnitude too small. We make up this difference by suggesting that vacuum energy is much higher still and that more degrees of freedom exist in physics beyond the Standard Model or that there is interaction energy between the modes.

      Delete
    5. Absolutely, we dont derive GR from QFT, but we expect to derive them both from some other source, dont we? We know for sure that both theories are effective/emergent/not-final.

      I will put my point differenty. We have in GR a constant describing gravitating properties (energy density) of the Vacuum. We have QFT describing gravitating properties (energy density) of the very same stuff - Vacuum. It is absolutely irrelevant that the latter is not measurable within the QFT, and the former not predictable from GR. It is irrelevant simply because both theories are expected to describe the very same physical reality.

      So, I am urged as a positivist to think that these two quantities which describe the very same physical property in two different theories must be related to each other in some very simple way - be equal to be precise. (that's what I so unfortunately called 'natural' ;) )

      A different question is that we might lack some additional info (from the same more general theory giving both GR and QFT) that would explain why the two quantities actually do describe two different properties of physical reality, aor otherwise should not be equal.

      Delete
    6. Ignat,

      "So, I am urged as a positivist to think that these two quantities which describe the very same physical property in two different theories must be related to each other in some very simple way - be equal to be precise. (that's what I so unfortunately called 'natural' ;) )"

      First of all, that you believe this should be the case isn't an argument, it is, to say the obvious a belief. Second, your belief is evidently wrong, so I don't know why you even bring it up.

      Delete
    7. Ignat, It seems that the GR and QFT values of the vacuum energy are pointing to two separate things. The GR value would be energy caused by some force of matter. The QFT value would possibly be pointing at something closer to the true energy density of the vacuum. I feel that the true energy density of the vacuum could be much larger than the GRCC^122 orders, possibly GRCC^122^100 orders. Just something I have been contemplating for a while.

      Delete
    8. Write up what you feel in a coherent manuscript and submit it for peer review or review amongst your colleagues.

      Delete
    9. Usually, you'll find that when challenged, the average professor has nothing to say. Putting you m.s. into the peer review system forces them to review it but they'll probably come back with b.s. on stuff they don't understand, suffer from cognitive dissonance or just plain envy. Modern profs have too much power and stultify progress in science. As Planck said, science proceeds one death at a time.

      Delete
    10. Sabine,
      So you dont 'believe' that two different theories describing (aiming to describe) the same specific property of the physical reality should give the same answer?

      And if this 'belief' is *evidently* wrong, what is the actual evidence for that, if you don't mind? =)

      Delete
    11. Peter,

      Yes, I would agree with that as a possible explanation - we need a new piece of theory to coincile two existing descriptions.

      My argument with Sabine is about the non observability of the QFT vacuum energy *in principle*. I state that the measurement of Lambda should [to the best of our current understanding] observe the QFT vacuum energy, and the fact that it does not does indicate a contrudiction between two theories. And the latter can indeed be resolved among the lines that you indicate in you comment.

      Delete
    12. Ignat,

      You are very confused. The vacuum energy in QFT is unobservable. There is no reason to think it is identical to the cosmological constant, which is a constant in a different theory. You seem to believe that they should be the same. I don't know why you think so. There is certainly no reason why two constants in two different theories -- the one observable, the other not -- should be the same.

      "And if this 'belief' is *evidently* wrong, what is the actual evidence for that, if you don't mind? =)"

      Seriously? You don't know that the QFT vacuum contribution does not agree with the measurement of the cosmological constant? You still don't know that naturalness arguments are just wrong, falsified by evidence, dead and out the window?

      I suggest that you read my book "Lost in Math".

      Delete
    13. All, these are the average professors I am talking about. Nobody quite knows what they've "discovered" but they seem to splash themselves all over social media like they matter. They're only capable of regurgitating facts but contribute nothing. They do indeed stand on the shoulders of giants and try to claim genius by association... but the science isn't falling for it.

      Delete
  35. Is anybody here familiar with the phrase, math doesn't lie?
    I'm not familiar with the language, but it seems to me that it has been used to spread a lot of foolish ideas.

    ReplyDelete
  36. If dark matter were slowing down light then the red shift would vary depending on where you looked since there is more dark matter in some places than in other places. The speed of light would change depending on time and place. It would be a mess.

    Unusual theories produce unusual observations. Its always good to ask if X were true how would it effect Y.

    ReplyDelete
  37. Dear Sabine,
    Our original thread got filled by a different discussion, so I will start a new one.

    I am sorry you picked up on the word I used (`naturally`), but my argument has nothing to do with 'naturalness' from your book, which I did read. 

    I do 'believe' that two theories describing the same (and the only one) physical reality must be compatible. If two theories (either by calculation, or by measurement) give a number for the same physical quantity, these two numbers must coincide irrespectively of their roles in each individual theory. 

    You can call it a belief, but I think it is just the transitivity of equality: if A=B and C=B, then A=C. I doubt we could talk science without the premises of this type of logic.

    The disagreement between Delta (let's call this way the QFT zeropoint energy) and Lambda is not evidence against this general statement, simply because this discrepancy can be resolved by simpler means (see below). 

    You state that 'Delta is unobservable'  and ' Lambda is constant of nature'. These two statements are false. To make it more clear, let me put them in more words and compare against the correct statements:
    (1) 'Delta is unobservable in principle whatever is the correct complete theory of the physical reality' vs 'Delta is unobservable within QFT'
    (2)'Lambda is constant of nature whatever is the correct complete theory of the physical reality' vs 'Lambda is a free constant within GR'
    The first statement in each pair is false (or unprovable if you prefer, thus non-scientific - we dont know the complete theory yet, neither we know if we will ever know it), the second statements are the true ones. And in the second form they are not a predicament to the following argument:

    To the best of our understanding of the two theories, Delta and Lambda do describe (should describe) the very same property of very same physical reality - energy density of a vacuum configuration. Thus, according to the aforementioned general statement, their values should be the same. The simplest explanation of the fact they are not - is the incompleteness of (our understanding of) the theories, and we do know they are not complete/correct!

    Summarizing, The GR and QFT do not live in two different physical realities, they describe the very same one (and stem from the same correct complete theory), and thus Delta and Lambda should coincide - they both (supposedly) describe the very same property of the reality. And the fact they do not coincide is simply one more argument to the incompleteness of our understanding of GR-QFT.


    Ignat,

    ReplyDelete
    Replies
    1. Ignat,

      You may not be aware that you are using an argument from naturalness, but that's what it is.

      "I do 'believe' that two theories describing the same (and the only one) physical reality must be compatible. If two theories (either by calculation, or by measurement) give a number for the same physical quantity, these two numbers must coincide irrespectively of their roles in each individual theory. "

      The matter under dispute is that these two numbers describe the same thing. They arguably do not.

      "To the best of our understanding of the two theories, Delta and Lambda do describe (should describe) the very same property of very same physical reality..."

      Again, you are just assuming what you want to show, and again, it's just wrong. Why do we have to discuss this?

      Delete
    2. 'The matter under dispute is that these two numbers describe the same thing.'

      Of course! that's exactly the question. But it is raised becasue Delta and Lambda do not coincide, not answered by it. While these two quantities by their construction and implied physical meaning should coincide, they do not for some yet unknown reason. And thus we conclude that most likely these two numbers do not describe the same thing indeed. So there is a discrepancy, contrary to what you state in the video, and the discrepancy does show (once again) that our current understanding of GR-QFT is wrong.


      As to your reference to naturalness, I cannot see your point, unfortunately. I dont assume that theories must look beautiful, all numbers be of a certain scale, etc. I only assume (together with you: A=C!) that the same quantity in two different theories describing the same reality must have the same value, and if it does not happen, I look for the reasons why.

      Delete
    3. It's quite simple - GR responds to absolutes in energy. If you have a theory (QFT) which predicts some energy constant that you can't measure by the usual differences of measurement in that theory, it ought to show up in GR and exert an absolute effect.
      If you get the magnitude all wrong, then modify how you insert your constant into GR AND THEN YOU WILL HAVE SOMETHING YOU CAN INFER FROM MEASUREMENT.
      Lambda IS the gravitational effect of QFT vacuum, if you do it right. It is directly observable otherwise neither theory makes sense.

      Delete
    4. "Lambda IS the gravitational effect of QFT vacuum"

      That's just wrong. GR knows nothing about the QFT vacuum vev. Lambda is a free constant in GR. Look, just imagine for a moment there were no quantum fields in the universe, nothing. Lambda would still be there.

      Delete
    5. Yeah and imagine there was no mass-energy too - then what would GR be doing. Have you any suggestions on what to do with either Lambda or QFT vacuum or are you just going to keep putting your hands over your ears and going na na na na? It's a problem to be solved or don't people solve problems these days?

      Delete
    6. Remi,

      Getting hostile now that you understand you are just wrong, huh?

      "Have you any suggestions on what to do with either Lambda or QFT vacuum"

      There is no problem with Lambda or the QFT vacuum. The problem is that physicists waste their time on a pseudoproblem.

      Delete
  38. Maybe you could do a song,
    .
    "Imagine there's no QFT vacuum
    It isn't easy but try(!)
    No meaningful physics either...
    A complete bl..dy lie.
    .
    Imagine all the wasted effort
    20th century physics has achieved...
    Yoo hoo
    .
    You might say it's all b.s.
    But just what else have you've got?
    I hope some day you'll see sense
    And start believing what's before your eyes"

    ReplyDelete
    Replies
    1. This comment has been removed by the author.

      Delete
    2. Please lay out your stall. You are saying, just as a mathematical edifice that GR permits no static solutions without that cursed constant...? The only thing to counter gravitational attraction then would be the "centrifugal force" of things orbiting BUT THAT IS NOT THE GENERAL CASE.
      In general, in GR, there are no static solutions.
      It can only be countered by Lambda, furthermore, Lambda doesn't have to allow for a static universe, but an expanding one, as Dr Hubble observed.

      Delete
    3. wtf are you even talking about? I have no time for this nonsense and will not approve further comments from you.

      Delete
  39. Please lay out your stall - say what you mean that "Lambda is a free constant". It was inserted to stop everything collapsing in on itself. It was later made to allow expansion of the universe too, as per observation.
    You are an educator, say what you mean.
    Then tell us how you can have GR without the vacuum of QFTs and how you prefer adding a mysterious constant to make GR work, rather than linking it to any other known physics (physics in a vacuum indeed!)
    I couldn't make myself clearer. Everyone here understands what I am saying. I could even bounce it off a few laurates I know.

    ReplyDelete
    Replies
    1. Lambda is a one of the two constants in GR. It has no fixed value, you determine it by measurement. That it's "free" means exactly this: It has no fixed value, hence you determine it by measurement, and that's the end of the story. You can add whatever else you want to add to a free constant, it'll still be a free constant. This can't possibly be so hard to understand.

      Delete
    2. Just like the constant of c in Maxwell's wave equation suspiciously bore a resemblance to the measured speed of light and he made the connection that light was electromagnetic. Hey, but it's not as though physicists get paid to link ideas up 🤣

      Delete
    3. Yeah, you can measure Lambda and it doesn't resemble anything. Congrats, you have just explained that you didn't understand a word I said.

      Delete
    4. It doesn't resemble anything because you're not looking hard enough.
      It's not the kind of constant that you can add a datum to or merely change units: you can be 30 feet above sea level or approximately 10 meters, or you set your datum at another town's height - you're 9 feet above some town or about 3 meters above it. You're "free" to do what you like with such a constant.
      Or am I missing your point?

      Delete
    5. Explain, please.
      The EFEs are a differential relation. Lambda is not some constant of integration, say like integrating a 1/r^2 force and getting a potential 1/r and setting where your datum of energy measurement is.
      You can measure it but that doesn't mean it is unrelated to something else.
      The datum of a 1/r potential is completely arbitrary.

      Delete
    6. Remi,

      I did not say Lambda is a constant of integration, stop inventing things I didn't say. I said it's a constant of nature. Its value is not determined in GR, it's a free constant. If you add any constant to it, it's still a free constant, end of story.

      This is really, really basic stuff. And of course that you *ADD* the qft contribution to Lambda is the reason physicists go on about allegedly unlikely cancellations etc etc. They go on about this because you need this argument to avoid the obvious: it doesn't matter if you add a constant to a free constant, it'll still be a free constant.

      Delete
    7. The constant isn't being added to. The constant is being *identified* as something else. Please stop implying that I am adding to it. The law of identity, A=A, that's really basic stuff.

      Delete
    8. "The constant isn't being added to. The constant is being *identified* as something else. Please stop implying that I am adding to it. The law of identity, A=A, that's really basic stuff."

      The QFT contribution comes from the T_\mu\nu term. It is arguably not the same as the Lambda term. If you postulate they're identical you just get a wrong theory.

      Delete
    9. I can lower a "tensored" Lambda to either put it on the RHS or LHS, like it conventionally is on the LHS. I see no different theory.

      Delete
    10. Look: k x trace Tmunu = rho - 3p can go on LHS as Lambda or RHS as SET.

      Delete
    11. Yeah, that's what my video is about, thank you so much for sharing your wonderful insights with us.

      Delete
  40. Dear Sabine,
    You keep repeating that Lambda is a free constant [within GR], and you are absolutely right. It is. But what you seem to forget is that GR is an effective [classical] theory and that this constant has a physical meaning!!! very specific physical meaning - energy density of vacuum.

    Trying to find an underlaying physics for apparent constants in effective theories is what physics is all about. Well, not all, you can also try to develope an independent theory which might (or might not) give a prediction for the value of the physical quantity in question. Qft seemingly doesn't work in this particular instance, thats all.

    You ask what whout happen to Lambda if no quantum fields existed? We would die to find another explanation for it, thats what would happen =) Bus since we do have quantum fields, we tried (and failed) to explain Lambda through their vev.

    (BTW, my last reply to you hasn't got published for some reason?)

    ReplyDelete
    Replies
    1. Ignat,

      "You keep repeating that Lambda is a free constant [within GR], and you are absolutely right."

      Of course it's right, and that's all you need to know. Everything else you add to this is entirely irrelevant.

      "But what you seem to forget is that GR is an effective [classical] theory and that this constant has a physical meaning!!! very specific physical meaning - energy density of vacuum."

      (a) We have no reason to think that GR is an effective theory. If you think so, then please derive it from something else and pick up your Nobel Prize.

      (b) As you say that Lambda is the "energy density" of the vacuum is an interpretation. I think this interpretation makes absolutely no sense. It must have occurred to you that it doesn't even have the dimension of an energy density. Or does it? What it actually is, if you look at the math, is the curvature of empty space.

      Even if you interpret it as an energy-density, it's still a constant you need to determine by measurement. To repeat this completely trivial point one more time: If you add a QFT contribution to it, that doesn't change anything about it being a free constant. It's completely irrelevant what you add.

      "Trying to find an underlaying physics for apparent constants in effective theories is what physics is all about."

      No it's not. The purpose of science is to describe nature. A constant of nature is the simplest description that you can possibly think of. People like you who think there's something wrong with a constant of nature are very confused about what science is to begin with.

      Delete
    2. Sabine: "(b) As you say that Lambda is the "energy density" of the vacuum is an interpretation. I think this interpretation makes absolutely no sense. It must have occurred to you that it doesn't even have the dimension of an energy density. Or does it? What it actually is, if you look at the math, is the curvature of empty space."

      Does not energy cause the curvature of space so Lambda should be a type of energy density? It is interesting the wiki page on Cosmological Constant does not mention curvature but repeatedly mentions Energy Density.

      " Ignat: "Trying to find an underlaying physics for apparent constants in effective theories is what physics is all about."
      Sabine: "No it's not. The purpose of science is to describe nature. A constant of nature is the simplest description that you can possibly think of. People like you who think there's something wrong with a constant of nature are very confused about what science is to begin with." "

      Sabine, you seem to be too willing to just accept Lambda as a "Constant Of Nature". That is true only because we do not know the cause of it and the underlying nature of it due to our incomplete theories. If Science is about description then we should be attempting to understand Lambda at a deeper level.

      Delete
    3. Peter Becher,

      "Does not energy cause the curvature of space so Lambda should be a type of energy density?"

      The energy density is in T_\mu\nu. If you set T_\mu\nu to zero, you have qua definition empty space. You contract the equations and find that \Lambda is the curvature of empty space. That's what the math says. Now, some people want to *interpret* this an an energy density of something. But there is no something now -- you have set the matter fields to zero.

      What is happening here is that a lot of people for some reason believe that empty space should be flat. But the math says clearly that doesn't have to be the case.

      "you seem to be too willing to just accept Lambda as a "Constant Of Nature". That is true only because we do not know the cause of it and the underlying nature of it due to our incomplete theories."

      That's false. I am merely talking about what is the status of the best theories that we currently have. I make no statements about what is "true". According to our best current theories, Lambda is a constant of nature and that's that. If you want to improve those theories, find a simpler explanation.

      Delete
    4. Sabine: "The energy density is in T_\mu\nu. If you set T_\mu\nu to zero, you have qua definition empty space. You contract the equations and find that find that \Lambda is the curvature of empty space."..."But there is no something now -- you have set the matter fields to zero."
      It would seem that Lambda is there because matter is there so space empty of matter cannot have curvature. Setting the right side to 0 would seem to make the left side equal to 0 also.

      Sabine: "If you want to improve those theories, find a simpler explanation."
      Working on it. Not a mathematical theory but a concept model. It will not be simpler since how can you get simpler that stating/measuring something into a constant of nature.

      Delete
    5. Peter,

      "t would seem that Lambda is there because matter is there"

      No it isn't. That's exactly what I'm saying. Lambda has nothing to do with the matter. It's a free constant that is *independent* of the matter. It's still there without any matter.

      Delete
    6. Sabine: "That's exactly what I'm saying. Lambda has nothing to do with the matter. It's a free constant that is *independent* of the matter. It's still there without any matter."

      Lambda is only there without matter because of the math and that it was measured and put in the GR equation. There is no way to "Measure" it without Matter.

      "Other scientists like Poincaré, Plank and Lorentz were very close to achieving the same results, but their unlingness to let go of tradition, impeded them to see the bigger picture, they just glimpsed parts of it, in the end, this willingness to let go of tradition, was what lead Einstein to see what they could not." by RelativityIsEasy

      We might need to let go of tradition and think outside the math box to move physics ahead.

      Delete
  41. What do you think of this interpretation of the cosmological constant?
    https://arxiv.org/abs/2102.09601

    ReplyDelete
  42. Sabine,

    "(a) We have no reason to think that GR is an effective theory."
    We actually do have - there must be a way to unify GR with QM, say for an electron. This new theory (yet unknown) will include GR as one of its limits, and it is in this sense that I called GR an effective theory. If you know how to solve contradictions between GR and QM without actually quantizing gravity, I am very keen to learn it (and you will get your Nobel prize for it as well :) 
     
    "(b) As you say that Lambda is the "energy density" of the vacuum is an interpretation. I think this interpretation makes absolutely no sense."

    I am sorry but it seems to me that now you contradict your own video where you state (without a hint of disagreement)
    "You can therefore interpret Lambda as the energy-density of the vacuum."
    and also later
    "What does the energy-density of vacuum have to do with the acceleration of the universe?"
    and finally 
    "Okay, so we have learned: vacuum has an energy-density, it’s a constant of nature [...]"
    which all very clearly imply that you agree with such an interpretation. If you actually do not, you should edit your video IMHO.


    But if we agree to interpret it as an energy-density, then you argue that "It's completely irrelevant what you add" [to Lambda].

    Well, this statement is not complete. Irrelevant to what specifically? I would say that it is very relevant indeed to the...physical interpretation of this constant! If Lambda is a [gravitating] energy density of vacuum, and we represent it as a sum of [fluctuating] energy density of vacuum (QFT vev) plus something else (so that the sum of these two terms equals the observed value), this 'something else' must be interpreted somehow. What is it? Anit-gravitating energy density of vacuum? or gnomes running around and creating centrifugal force? (just kidding)

    So, when you state that "If you add a QFT contribution to it, that doesn't change anything about it being a free constant" it is a false statement. What does not change is its value, you can adjust for it indeed, but what does change is its physical interpretation.

    And 'describing nature' is not just about playing with 'free' numbers as "things in and by themselves". It is about *physically interpreting* the numbers coming from mathematical models, attributing these numbers to certain properties of physical reality.

    Since we do not have (to the best of my personal knowledge) a slightest hint of a physical interpretation of this new term (equal to observed Lambda minus QFT vev) we speak of a contradiction, or discrepancy between QFT prediction and GR observation.

    ReplyDelete
    Replies
    1. Ignat,

      "which all very clearly imply that you agree with such an interpretation. If you actually do not, you should edit your video IMHO."

      That I say you *can* interpret it in this way doesn't mean I think that it's useful. I had as a matter of fact several paragraphs in the original script explaining why this makes no sense, but I took them out because they were off-topic. I will certainly not take advise from some random commenter on my blog how to improve my videos.

      "So, when you state that "If you add a QFT contribution to it, that doesn't change anything about it being a free constant" it is a false statement. What does not change is its value, you can adjust for it indeed, but what does change is its physical interpretation."

      Now you're just talking nonsense. I have no patience for this sorry. Do some reading.

      Delete
  43. PS
    "People like you who think there's something wrong with a constant of nature are very confused about what science is to begin with."

    Well, I dont think there is anything wrong with constants of nature. Can you actually *prove* that Lambda is a constant of nature and not just a free parameter in a particular physical theory?

    ReplyDelete
    Replies
    1. You can only prove mathematical relations. This comment just further documents that you don't understand how science works in the first place.

      Delete
    2. "You can only prove mathematical relations."
      That's exactly my point, you cannot prove it is a constant of nature. So, using the satetement that it is a consnat of nature as an argument is a false argument.

      I must confess, I got tired of your personal remarks/attacks instead of replying the actual arguments. Thats appalling.

      Delete
    3. "That's exactly my point, you cannot prove it is a constant of nature. So, using the satetement that it is a consnat of nature as an argument is a false argument."

      Hahaha. Let me see. According to you one isn't allowed to call anything a constant of nature because it hasn't been mathematically proved. Neither is one allowed to call the internet the internet because that hasn't been mathematically proved. One is also not allowed to say that Washington is in the United States because that hasn't been mathematically proved. And so on. Not that anyone needed any more evidence that you're way out of your depth.

      "I must confess, I got tired of your personal remarks/attacks instead of replying the actual arguments. Thats appalling."

      You have no argument. You are "insulted" merely because I point out you are wrong. I have never personally attacked anyone. I don't even know who you are, so how could I. You're just some random commenter who wastes my time.

      Delete
    4. Ignat wrote and you came back:
      (b) As you say that Lambda is the "energy density" of the vacuum is an interpretation. I think this interpretation makes absolutely no sense. It must have occurred to you that it doesn't even have the dimension of an energy density. Or does it? What it actually is, if you look at the math, is the curvature of empty space.
      .
      And you are missing the point (again) of algebraic manipulation and taking Lambda onto the RHS and putting it into the stress-energy tensor:

      doi:10.20944/preprints201901.0113.v4

      I like your music videos. I think you're good at that. I am literally one degree of separation from Simon Cowell (UK and USA X-factor) via my nieces. If you can come up with a hit song, I could have it pitched to Gaga or Beyonce. This might be your true calling.

      Delete
    5. "And you are missing the point (again) of algebraic manipulation and taking Lambda onto the RHS"

      Let me see. You actually think I do not know that you can substract a term from one side of the equation and add it to the other side, yes? And that despite the fact that the term, urm, is on the right side on the thumb of my video? And you also think that somehow moving a term to another side of the equation makes any difference to the equation? Seriously?

      Delete
    6. Of course, if you can *identify* it with something else/give meaning to it.
      (Simon Cowell is waiting, it only takes one hit record and you're a star! Give up the day job... Sun yourself on the beach).

      Delete
    7. Hi Remi,

      'I am literally one degree of separation from Simon Cowell (UK and USA X-factor)'

      I personally am not worth a packet of quantum peanuts to science but I have had the pleasure of meeting and/or becoming acquainted with several live/recording performers, producers DJs and music radio presenters, and assorted other in luminaries of the Sydney and Australian live and recorded music scenes. I also have some access to people who work in Artists and Repetoire, promotion, marketing and the like, inluding for award winning bands and solo artists over the last few years.

      I have met these people directly through attending performaces, workshops, attending industry events, going to pub karaoke and friends' gigs, and attending a university-level college with lecturers who are musicians, producers and electronica performers in their own right, including a founder of a major record label in Australia (Chris Hancock of Elefant Traks, which releases famous Australian hip-hop artists). Give me a couple or 3 years of study and working to build my discography and career, Dr. Hossenfelder ought be able to utilise me as a networking contact and musical colleague, if she wants.

      If I asked the right people to pass me along, I could make my way to Simon Cowell and his like, I reckon, whether Mr. Cowell would give me the time of day is another matter.

      My best mate and ex-housemate is handily partnered with a singer who fronts 2 bands and knows a LOT of performers and industry people. (She's a darling.)

      I'm also happy to count Dr. Hossenfelder as a musical acquaintance of sorts; if she wants to collaborate, network or come to Sydney for music she's welcome to hit me up if she thinks it's worth her while.

      If you want a good night out in Sydney, hit me up. I know great places to go for a night out, including some great synth-pop, experimental and dance venues. :)

      Delete
    8. Spent a couple hours going through Remi’s online persona. It was quite entertaining, to say the least.

      Delete
    9. Sigh, if only... I heard a rumour London is going into another lockdown. Plenty time to read though! I shall peruse what Sabine and others are cooking, just out of interest!

      I'm hoping Sabine, if she hasn't done already, that she could do a video on the constants of nature: c, h, Alpha, Feignbaum - the real mysterious stuff... and I don't mean Lambda (I believe that one is ultimately a function of c, h and Alpha but she'll skin me)!!!

      One on Decoherence Theory/Super-determinism (Block Universe)/Everett's Many Worlds/Bohm's Pilot Wave etc. and which she thinks are true and which she thinks are b.s. and why.

      Maybe she (keep referring to her in the 3rd person, don't wish to be rude or to make her paranoid) could do one Second Law Exception theory (a la Sheehan/Capek), that would be fun.

      Delete
    10. Yeah, my nieces did very well. I am very proud of them. They came 2nd on X-Factor UK. They're part of "Lemonade" (youtube). It's not really my kind of music but good luck to 'em. I think we've digressed a little too far, so I'll keep it short and sweet and back to the physics.

      Delete
    11. Sincere congratulations to them, Remi. I hope they are enjoying continued success. My live public singing performances so are only pub karaoke.

      Dr. Hossenfelder is quite literally my most famous acquaintance (of sorts). I mean, she's been in broadcast discussions with Roger Penrose, multiple Seans, Brians and Stephens, etc. (more people than I can remember off the top of my head,) She seems to be rather well-recieved. That's one heck of a roster to appear with. I should be asking her to make *me* famous.

      Delete
    12. Good for you but oh dear! let's stick to physics and short to-the-point answers on these boards! Fame means nothing, I'm only interested in the physics and not TV professors!

      Delete
    13. Sabine, if anything, is a scholar of physics. She may not be the most creative or have made any real discoveries but physics professors spend their time teaching and reading and they can have a library-like mind to research not just in their field but overall. You have a great chance here to pick her brains and if she sends you off to read something, it is probably very apposite. I wouldn't waste it on celeb tittle tattle.

      Delete
    14. Remi,

      Sabine, if anything, is a researching physicist; I know that much. If she answers my questions I'm well pleased even if she tells me I need to go look at the math.

      All sorts of things get mentioned here but we circle back to the topics at hand.
      I'm one of the people who follow Dr. Hossenfelder because of the excellently entertaining videos she makes then we've found out about the rest of her research, interviews/panels and publishing career, so we hold her in high esteem. We also contribute to discussions here, and learn. Dr. H is hopefully used us by now.

      Delete
  44. Sabine,
    Where did I say that you cannot call it a constant of nature? you misinterpret completely what I said. I said that just calling something a contant of nature is a not an argument per se.

    As to the personal remarks, when you call repeatedly other side's arguments a nonsense and state that the author 'does not understand physics' (instead of actually analysing and replying the arguments) it is indeed a personal remark. In a scientifiс discussion it is not enough to simply `point out it is wrong`, you need to prove it is wrong.

    I do simpathise with you facing all very different kinds of commenters online, but if you choose to defend science online your policies (on politness and constructivity of your arguments) should be stricter. (oh, yes that is just another advice from a random commenter, no need to reply please)

    Would it change my randomnes? probably not, but just in case https://scholar.google.com.br/citations?user=m2EotE8AAAAJ&hl=en

    ReplyDelete
    Replies
    1. Ignat,

      "when you call repeatedly other side's arguments a nonsense and state that the author 'does not understand physics' (instead of actually analysing and replying the arguments) it is indeed a personal remark."

      No it is not. A personal remark, needless to say, is a remark about the person.

      Also, what you say is patently untrue. This very comment section is proof that I have as a matter of fact repeatedly answered to your nonsense. And that was my last answer.

      Delete
    2. Ignat,

      “there must be a way to unify GR with QM, say for an electron. This new theory (yet unknown) will include GR as one of its limits”

      Indeed.

      “If you know how to solve contradictions between GR and QM without actually quantizing gravity, I am very keen to learn it”

      1. We know QM is incomplete (assuming locality) – the EPR argument, so we can be almost sure that the current understanding of the electron and other particles is not the correct one. QM is just a statistical approximation.

      2. The failure to “quantize” gravity even after decades of working by very smart physicists is evidence that it probably cannot be done.

      3. Models that describe QM as a statistical limit of a classical theory started to emerge:

      Probabilistic cellular automata for interacting fermionic quantum field theories

      http://128.84.21.203/pdf/2106.15517

      Fast Vacuum Fluctuations and the Emergence of Quantum Mechanics

      https://arxiv.org/abs/2010.02019

      So, we have no good reasons to believe that making GR “quantum” is a must.

      Regarding QFT vev, an expectation value is usually interpreted as a result of some measurement, not an objective property of the system. That result makes sense only in the context of that measurement. I have yet to see a good argument for why we should make an exception for the vev.

      We actually have some evidence that the concept of vacuum fluctuations is not real, since it’s possible to explain Casimir forces without it:

      The Casimir Effect and the Quantum Vacuum

      https://arxiv.org/pdf/hep-th/0503158.pdf

      So, it might be the case that QED does not say anything about vacuum and no contradiction with GR comes from this.

      Delete
    3. Couldn't it be done via the mass? Mass is at the root of contraction and dilation in Relativity. If there is field (Higgs is part of it) that describes mass, quantise that.
      doi:10.20944/preprints201903.0056.v1

      Delete
  45. Dear Andrei,

    Maybe you find interesting also the work on emergent quantum mechanics,

    https://arxiv.org/pdf/1402.5070.pdf

    This work contains a "derivation" of the weak equivalence principle from a statisical framework for subquantum mechanics, contains a mechanism for the collapse of the wave function, contains a mechanism that explains the non-locality of quantum phenomena.

    It does not deal with the cosmological constant, however.


    ReplyDelete
    Replies
    1. Topolino,

      That paper proposes two time dimensions. I have no idea what is the meaning of that.

      Delete
    2. There are several ways to understand a two dimension time. The first and more conventional refers to dynamical systems where there are fast variables and slow variables. This is rather typical in classical dynamics. Then there is a two-dynamical physics in the context of M-theory, developed T. Banks 20 years ago. This is a geometric concept: spacetime is 11-dimensional with signature 11-2 etc...

      What this other author is proposing is that the notion of time is really two dimensional: one parameter describes the dynamics of the fast variables at the fundamental scale; the second parameter is an emergent time(s) parameter and is associated with any type of regular phenomenon used to parametrize evolution in classical or quantum models. The difference between this notion of two-dimensional time and the one used in dynamical systems is that they are un-related (in classical dynamics, there is a diffeomorphism between the time parameters; it is just confortable to consider two time parameters, depending on the variables one is considering).

      In "philosophy" there was some ideas on two-dimensional time, but I do not understand them enough.

      Delete
    3. Sorry,

      it was not T. Banks, but I. Bars who has been investigating two dimensional physics already for many years:

      http://physics.usc.edu/~bars/twoTph.htm

      Delete
  46. Ether, Dirac Sea, quantum field vacuum and dark xyz...
    Perplexity and phenomenology remoteness are "replaced" by postulated virtual particles and postulated interactions. Since this cannot succeed as a consistent phenomenolgy, it requires many free parameters, variable coupling constants and "wild substructuring" as complexity progresses. See for example: One exchange particle (photon) of the QED quickly “becomes” eight gluons (QCD), which then have to be equipped with additional quantum numbers.
    In the case of the postulated ether, one obtained a carrier medium that conducts light, in the case of the (negative) Dirac Sea, one obtained a medium in the vacuum that is full of postulated negative energy particles that cause the fluctuations in the vacuum. In both cases, the postulated medium can neither be justified nor proven. "Funnily enough", Standard Model supporters reject(ed) the unobserved ether and then introduce the equally undetectable dark energy and dark matter. Viewed soberly, one belief concept is simply replaced by another belief concept.
    Starting from the quantum field vacuum, one of the literally great difficulties is that energy must be assumed in the vacuum, which "drags" its effects into GR (General Relativity). The question arises in connection with the cosmological constant: Is the zero-point energy real? Or will it “evaporate” one day, just as the ether and Dirac Sea did before.

    ReplyDelete

COMMENTS ON THIS BLOG ARE PERMANENTLY CLOSED. You can join the discussion on Patreon.

Note: Only a member of this blog may post a comment.